The length of a rectangle is 4 in longer than its width.
If the perimeter of the rectangle is 32 in, find its area.

Answers

Answer 1

Answer:

60 sq in

Step-by-step explanation:

Perimeter = 2l + 2w

If l = w+4

Perimeter = 2(w+4) + 2w

Perimeter = 4w+8

32 = 4w + 8

24 = 4w

6 = w

If w = 6, l = 6+4 = 10

Area = l * w

Area = 10 * 6

Area = 60


Related Questions

A car insurance company has determined that6% of all drivers were involved in a car accident last year. If14drivers are randomly selected, what is the probability of getting at most 3 who were involved in a car accidentlast year

Answers

Answer:

[tex]P(x \le 3) = 0.9920[/tex]

Step-by-step explanation:

Given

[tex]p = 6\%[/tex] --- proportion of drivers that had accident

[tex]n = 14[/tex] -- selected drivers

Required

[tex]P(x \le 3)[/tex]

The question is an illustration of binomial probability, and it is calculated using:

[tex]P(x ) = ^nC_x * p^x * (1 - p)^{n-x}[/tex]

So, we have:

[tex]P(x \le 3) = P(x = 0) +P(x = 1) +P(x = 2) +P(x = 3)[/tex]

[tex]P(x=0 ) = ^{14}C_0 * (6\%)^0 * (1 - 6\%)^{14-0} = 0.42052319017[/tex]

[tex]P(x=1 ) = ^{14}C_1 * (6\%)^1 * (1 - 6\%)^{14-1} = 0.37578668057[/tex]

[tex]P(x=2 ) = ^{14}C_2 * (6\%)^2 * (1 - 6\%)^{14-2} = 0.15591149513[/tex]

[tex]P(x=3 ) = ^{14}C_3 * (6\%)^3 * (1 - 6\%)^{14-3} = 0.03980719024[/tex]

So, we have:

[tex]P(x \le 3) = 0.42052319017+0.37578668057+0.15591149513+0.03980719024[/tex]

[tex]P(x \le 3) = 0.99202855611[/tex]

[tex]P(x \le 3) = 0.9920[/tex] -- approximated

Jack’s backpack weighs 15 pounds. Fernando’s backpack weighs less than Jack’s. Which graph shows how much Fernando’s backpack can weigh?

Answers

Answer:

A

Step-by-step explanation:

c and d out of the question

b has its circle filled in meaning it could be 15lbs, which it's not

A correct answer by default

Answer:b

Step-by-step explanation: it has a filled in diamond which mean it's that...

Simplify the following expression

Answers

Answer:

[tex]\frac{98p^{6}}{q}[/tex]

Step-by-step explanation:

Distribute the exponents

[tex](\frac{(7^{-2}p^{-6}q^{-8})}{2q^{-9}} )^{-1}[/tex]

[tex](\frac{q}{98p^{6}} )^{-1}[/tex]

Distribute the -1

[tex]\frac{98p^{6}}{q}[/tex]

Can someone do #4 and #5

Answers

Answer:

First, find two points on the graph:

(x₁, y₁) = (0, 2)(x₂, y₂) = (2, 8)

Slope = [tex]\frac{y_{2}-y_{1} }{x_{2}-x_{1}} = \frac{8-2}{2-0} =\frac{6}{2}=3[/tex]

16 + (-3) = 16 - 3 = 13

According to this diagram, what is tan 62°?
62°
17
18
280
90°
15
O A.
8
17
OB.끝
O c. 1
8
15
D.
15
8
O
E.
17
15
F.
15
17

Answers

Answer:

15/8

Step-by-step explanation:

tan(62)=P/B

tan(62)=15/8

5 A machine puts tar on a road at the rate of 4 metres in 5 minutes.
a) How long does it take to cover 1 km of road
b) How many metres of road does it cover in 8 hours?​

Answers

Answer:

5 a) Total = 20.83 hrs = 20 hrs and 50 mins  (1250mins total)

5 b) Total = 96 meters. = 0.096km in 8 hrs.

Step-by-step explanation:

1km = 1000 meters

5 mins = 4 meters

1000/4 = 250 multiplier

250 x 5mins = 1250 minutes

1250/60 = 20hrs + 50 minutes

50 / 60 =  0.83 = 20.83hrs

b)  8 hrs = 8 x 60 = 480 minutes

480/5 = 24 multiplier of 4 meters

24 x 4 = 96 meters

Which two shapes make up the digital camera below?

Answers

Rectangular prism and cylinder

Rectangular prism and cylinder make up a camera.

What is rectangular prism and cylinder?

A cube is a rectangular prism with all of its sides being the same length, a triangular prism has a triangle as its base, and a rectangular prism has a rectangle as its foundation. Another form of right prism that has a circle as its basis is a cylinder.

A rectangular prism includes a total of 6 faces, 12 sides, and eight vertices. Like a cuboid, it contains three dimensions- the base width, the height, and the length. The top and base of the rectangular prism exist rectangular. The pairs of opposite faces of a rectangular prism exist as identical or congruent.

A cylinder contains traditionally been a three-dimensional solid, one of the most essential curvilinear geometric shapes. Geometrically, it includes been regarded as a prism with a circle as its base.

Hence, Rectangular prism and cylinder make up a camera.

To learn more about rectangular prisms and cylinders refer to:

https://brainly.com/question/15594686

#SPJ2

This graph represents which of these expressions?

Answers

Answer:

x > 43

Step-by-step explanation:

Open circle at 43, which means it is not equal to 43

Line goes to the right, which means x is greater than

x > 43

Answer:

B

Step-by-step explanation:

2(4×+2)=10
[tex]6 \times + 4 = 10 \\ \\ 6 \times = 10 - 4 \\ \\ 6 \times = 6 \\ \\ [/tex]
that is the answer

Answers

Answer:

x = 3/4

Step-by-step explanation:

2(4x + 2) = 10           Remove the brackets

8x + 4 = 10               Subtract 4 from both sides

8x = 6                       Divide by 8

x = 6/8

x = 3/4

Check

2(4*3/4 + 2) =?10

2( 3 + 2) = 10

2*5 = 10

10 = 10

Your parents deposit 2 50-dollar bills at the bank.
How much money did they deposit?

Answers

$100 is what they deposited!!

Answer: $100

Step-by-step explanation:

Considering 50 + 50 = 100

This means that the amount of money is $100

Complete the table for the given rule.
Rule: y is 0.75 greater than x
x y
0
3
9

Answers

The complete table is

x    y

0    0.75

3     3.75

9     9.75

What is equation?

An equation is a mathematical statement that is made up of two expressions connected by an equal sign.

What is substitution?

Substitution means putting numbers in place of letters to calculate the value of an expression or equation.

According to the given question.

We have values of x.

Also, one rule that y is 0.75 greater than x.

So, we have a equation for finding the value of y  i.e.

[tex]y = x + 0.75..(i)[/tex]

For finding the value of y

At x = 0, substitute x = 0 in equation (i)

[tex]y = 0 + 0.75\\\implies y = 0.75[/tex]

At x = 3, substitute x = 3 in equation (i)

[tex]y = 3+0.75\\\implies y = 3.75[/tex]

At x = 9, substitute x = 9 in equation (i)

[tex]y = 9+0.75\\\implies y = 9.75[/tex]

Hence, the complete table is

x    y

0    0.75

3     3.75

9     9.75

Find out more information about equation and substitution here:

https://brainly.com/question/10852714

#SPJ2

Write the additive inverse of each of the following rational numbers:

Answers

Answer:

(1)-3/4, (2) 5/21, (3)4/43

HELPPPPP ASP PLZZZZZ

Answers

Answer:

[tex](f-g)(x)[/tex]

[tex]f(x)-g(x)[/tex]

[tex]x^{2} -6x-27-x+9[/tex]

[tex]x^{2} -7x-18[/tex]

----------------------

[tex](f*g)(x)[/tex]

[tex]=f(x)g(x)[/tex]

[tex](x^{2} -6x-27)(x-9)[/tex]

[tex]=x^{3} -15x^{2}+27x+243[/tex]

----------------------

[tex]\frac{f}{g} (x)[/tex]

[tex]\frac{x^{2} -6x-27}{x-9}[/tex]

[tex]\frac{(x-9)(x+3)}{x-9}[/tex]

[tex]x+3[/tex]

-----------------------

[tex](f+g)(x)[/tex]

[tex]f(x)+g(x)[/tex]

[tex]=x^{2} -6x-27+x-9[/tex]

[tex]=x^{2} -5x-36[/tex]

------------------------

OAmalOHopeO

------------------------

Im new to this app!
And im looking for help!!
Please help ASAP!!!
Please!!!!

Answers

y=x²-10x-7

a>0 so we will be looking for minimum

x=-b/2a=10/2=5

y=25-50-7=-32

Answer: (5;32)

Find the equation of the linear function represented by the table below in slope-intercept form.

Answers

Answer:

y=-4x-5

Step-by-step explanation:

The slope of the line is - 4, the equation of line is y=-4x-5

John runs a computer software store. Yesterday he counted 140 people who walked by the store, 63 of whom came into the store. Of the 63, only 25 bought something in the store.
(a) Estimate the probability that a person who walks by the store will enter the store. (Round your answer to two decimal places.)
(b) Estimate the probability that a person who walks into the store will buy something. (Round your answer to two decimal places.)
(c) Estimate the probability that a person who walks by the store will come in and buy something. (Round your answer to two decimal places.)
(d) Estimate the probability that a person who comes into the store will buy nothing. (Round your answer to two decimal places.)

Answers

Answer:

.................

Step-by-step explanation:

............

a. 45%
b. 39.68%
c. 17.86%
d. 60.32%

look at the image for the question

Answers

It’s 16
You would multiply height (4) by the base (length(2) x width(2)=4) which is 16

1. A helicopter is at a position from two VORS (VHF Omnidirectional
Radio Range, an aircraft navigation system operating in the VHF band -
not covered in chapter) as in the diagram shown below. Given the angles
shown, find the third angle.
Helicopter
74.0°
66.0°
VOR
VOR

Answers

The position of the helicopter and the two VORs forms a triangle and the third angle formed by these three entities is 40 degrees

The diagram is not shown; however, the question can still be answered.

The given angles are:

[tex]\theta_1 = 74.0^o[/tex]

[tex]\theta_2 = 66.0^o[/tex]

Represent the third angle as [tex]\theta_3[/tex]

The helicopter and the 2 VORs form a triangle.

So, we make use of the following theorem to calculate the third angle

[tex]\theta_1 + \theta_2 + \theta_3= 180^o[/tex] ---- sum of angles in a triangle

Substitute known values

[tex]74.0^o + 66.0^o + \theta_3= 180^o[/tex]

[tex]140.0^o + \theta_3= 180^o[/tex]

Collect like terms

[tex]\theta_3= 180 -140.0^o[/tex]

[tex]\theta_3= 40^o[/tex]

Hence, the third angle is 40 degrees.

Learn more about angles in a triangle at:

https://brainly.com/question/14780489

Find the area of the figure. (Sides meet at right angles.)

Answers

Answer:

56

Step-by-step explanation:

A=(3*4)+(4*(4+3+4))=56

Find an upper bound for E(h) the error of the machine approximation of the two-point forward difference formula for the first derivative and then find the h corresponding to the minimum of E(h).

The two-point forward difference formula for f'(x) is:_________

Answers

Answer:

I doubt it is not going to be a great

Please Help!! Whoever helps and gets it correct gets Brainliest and 5 star rating!!

Answers

Answer:

the reasoning states that "all the numbers begin with a 7 or an 8"

however this is not accurate as they can be in different placements

which can make a big difference in the total estimate.

for example:

the number could've been an 8, or an 80

they both begin with an 8

however have totally different values and could have messed up the total estimated number.

hope this helps :D

Seven and one-half foot-pounds of work is required to compress a spring 2 inches from its natural length. Find the work required to compress the spring an additional 3 inch.

Answers

Answer:

Apply Hooke's Law to the integral application for work: W = int_a^b F dx , we get:

W = int_a^b kx dx

W = k * int_a^b x dx

Apply Power rule for integration: int x^n(dx) = x^(n+1)/(n+1)

W = k * x^(1+1)/(1+1)|_a^b

W = k * x^2/2|_a^b

 

From the given work: seven and one-half foot-pounds (7.5 ft-lbs) , note that the units has "ft" instead of inches.   To be consistent, apply the conversion factor: 12 inches = 1 foot then:

 

2 inches = 1/6 ft

 

1/2 or 0.5 inches =1/24 ft

To solve for k, we consider the initial condition of applying 7.5 ft-lbs to compress a spring  2 inches or 1/6 ft from its natural length. Compressing 1/6 ft of it natural length implies the boundary values: a=0 to b=1/6 ft.

Applying  W = k * x^2/2|_a^b , we get:

7.5= k * x^2/2|_0^(1/6)

Apply definite integral formula: F(x)|_a^b = F(b)-F(a) .

7.5 =k [(1/6)^2/2-(0)^2/2]

7.5 = k * [(1/36)/2 -0]

7.5= k *[1/72]

 

k =7.5*72

k =540

 

To solve for the work needed to compress the spring with additional 1/24 ft, we  plug-in: k =540 , a=1/6 , and b = 5/24 on W = k * x^2/2|_a^b .

Note that compressing "additional one-half inches" from its 2 inches compression is the same as to  compress a spring 2.5 inches or 5/24 ft from its natural length.

W= 540 * x^2/2|_((1/6))^((5/24))

W = 540 [ (5/24)^2/2-(1/6)^2/2 ]

W =540 [25/1152- 1/72 ]

W =540[1/128]

W=135/32 or 4.21875 ft-lbs

Step-by-step explanation:

Round 573.073 to the greatest place

Answers

Answer:

574

Step-by-step explanation:

To round a two-digit number to the nearest ten, simply increase it or decrease it to the nearest number that ends in 0: When a number ends in 1, 2, 3, or 4, bring it down; in other words, keep the tens digit the same and turn the ones digit into a 0

Hope this helps <3

if U>T, R>Q, S>T and T>R, which of the following is TRUE?
1. S>Q
2. U > S
3.U > R​
A. 1 only
B. 2 only
C. 1 and 2
D. 2 and 3

Answers

Answer:

C. 1 and 2

Step-by-step explantation:

First, i would order them as U>T, T>R, R>Q, S>T

we can rewrite them as

U>T>R>Q,

now adding S, we get U>S>T>R>Q,

so U>S

We can also rewrite all of them as inequalities:

U-T>0

T-R>0

R-Q>0

S-T>0

Add R-Q and T-R

(R-Q)+(T-R)>0

-Q+T>0

T>Q, but because S>T we can say S>Q

Paul can install a 300-square-foot hardwood floor in 18 hours. Matt can install the same floor in 22 hours. How long would it take Paul and Matt to install the floor working together?
4 hours
9.9 hours
13.2 hours
30 hours

Answers

Answer:

9.9 hours

Step-by-step explanation:

The formula to determine the time together is

1/a+1/b = 1/c  where a and b are the times alone and c is the time together

1/18 + 1/22 = 1/c

The least common multiply of the denominators is 198c

198c(1/18 + 1/22 = 1/c)

11c+ 9c = 198

20c = 198

Divide by 20

20c/20 =198/20

c =9.9

Answer:

B - 9.9 hrs

Step-by-step explanation:

took the test.

8.9 x 10^3 in standard notation

Answers

Answer:

that is n standard notation mah frand

8.9 × 10^3 being scientific notation of " 8900 "

[tex]\huge\text{Hey there!}[/tex]

[tex]\large\textsf{8.9}\times\large\textsf{10}^\mathsf{3}\\\\\mathsf{10^3}\\\mathsf{= 10\times10\times10}\\\mathsf{= 100\times10}\\\mathsf{= \bf 1,000}\\\\\large\textsf{8.9}\times\large\textsf{1,000}\\\\\large\textsf{= \bf 8,900}\\\\\\\boxed{\boxed{\huge\text{Answer: \boxed{\underline{\underline{\bf 8,900}}}}}}\huge\checkmark[/tex]

[tex]\huge\text{Good luck on your assignment \& enjoy your day!}[/tex]

~[tex]\boxed{\huge\text{}\boxed{\frak{Amphitrite1040:)}}}[/tex]

find the missing side of the triangle

Answers

Answer:

x = 34

Step-by-step explanation:

Pytago:

x[tex]30^{2} + 16^{2} = x^2\\x = \sqrt{30^2 + 16^2} \\x = 34[/tex]

Find an equation of the plane orthogonal to the line
(x,y,z)=(0,9,6)+t(7,−7,−6)

which passes through the point (9, 6, 0).

Give your answer in the form ax+by+cz=d (with a=7).

Answers

The given line is orthogonal to the plane you want to find, so the tangent vector of this line can be used as the normal vector for the plane.

The tangent vector for the line is

d/dt (⟨0, 9, 6⟩ + ⟨7, -7, -6⟩t ) = ⟨7, -7, -6⟩

Then the plane that passes through the origin with this as its normal vector has equation

x, y, z⟩ • ⟨7, -7, -6⟩ = 0

We want the plane to pass through the point (9, 6, 0), so we just translate every vector pointing to the plane itself by adding ⟨9, 6, 0⟩,

(⟨x, y, z⟩ - ⟨9, 6, 0⟩) • ⟨7, -7, -6⟩ = 0

Simplifying this expression and writing it standard form gives

x - 9, y - 6, z⟩ • ⟨7, -7, -6⟩ = 0

7 (x - 9) - 7 (y - 6) - 6z = 0

7x - 63 - 7y + 42 - 6z = 0

7x - 7y - 6z = 21

so that

a = 7, b = -7, c = -6, and d = 21

An equation of the plane orthogonal to the line 7x - 7y - 6z = 21.

The given line is orthogonal to the plane you want to find,

So the tangent vector of this line can be used as

The normal vector for the plane.

The tangent vector for the line is,

What is the tangent vector?

A tangent vector is a vector that is tangent to a curve or surface at a given point.

d/dt (⟨0, 9, 6⟩ + ⟨7, -7, -6⟩t ) = ⟨7, -7, -6⟩

Then the plane that passes through the origin with this as its normal vector has the equation

⟨x, y, z⟩ • ⟨7, -7, -6⟩ = 0

We want the plane to pass through the point (9, 6, 0), so we just

translate every vector pointing to the plane itself by adding ⟨9, 6, 0⟩,

(⟨x, y, z⟩ - ⟨9, 6, 0⟩) • ⟨7, -7, -6⟩ = 0

Simplifying this expression and writing it in standard form gives

⟨x - 9, y - 6, z⟩ • ⟨7, -7, -6⟩ = 0

7 (x - 9) - 7 (y - 6) - 6z = 0

7x - 63 - 7y + 42 - 6z = 0

7x - 7y - 6z = 21

So that, a = 7, b = -7, c = -6, and d = 21.

To learn more about the equation of plane visit:

https://brainly.com/question/1603217

State if the scenario involves a permutation or a combination. Then find the number of possibilities.

A team of 15 basketball players needs to choose two players to refill the water cooler.

Permutation/Combination:

Answer:

Answers

Answer:

Permutation ; 210 ways

Step-by-step explanation:

Permutation and combination methods refers to mathematical solution to finding the number of ways of making selection for a group of objects.

Usually, selection process whereby the order of selection does not matter are being treated using permutation, while those which takes the order of selection into cognizance are calculated using combination.

Here, selecting 2 players from 15 ; since order does not matter, we use permutation ;

Recall :

nPr = n! ÷ (n - r)!

Hence,

15P2 = 15! ÷ (15 - 2)!

15P2 = 15! ÷ 13!

15P2 = (15 * 14) = 210 ways

Reason Can you subtract a positive integer from a positive integer
and get a negive result? Explain your answer.

Answers

Answer:

No

Step-by-step explanation:

No matter the situation, when you multiply a negative by a negativeyou get a positive and a positive by a positive you get a positive. but if its two different like a negative and a positive then its NEGITIVE.

let's say you have 23 and you're multiplying by 2.

It's always increasing so it doesnt ever reach the negitive numbers.

Other Questions
Which inequality describes this graph? Cuando el pistn tiene un volumen de 2x10^-4 m^3, el gas en el pistn est a una presin de 150 kPa. El rea del pistn es 0.00133 m^2. Calcular la fuerza que el gas ejerce sobre el embolo del pistn. Use the function below to find F(-4).F(x) = 2^x is what a drug does to your body. O a) Irrigation b) Pharmacodynamics c) Excretion d) Pharmacokinetics Mrs. Gomez has two kinds of flowers in her garden. The ratio of lilies to daisies in the garden is 5:2If there are 20 lilies, what is the total number of flowers in her garden? Cells dispose of large waste molecules through a process calledosmosis.diffusion.exocytosis.endocytosis. If a and b are positive numbers, find the maximum value of f(x) = x^a(2 x)^b on the interval 0 x 2. A system must include _______.A.an input, an output and a process.B.something to do with a form of transport.C.a microprocessor.D.fuel, water and electricity. Which of the following terms is the lifecycle of a living cell?A. g1B. cell cycleC. interphaseD. mitosis How would a Kantian deontologist evaluate lying? a.) It is morally wrong because if everyone did it, nobody could be trusted anymore. b.) It is morally wrong because a person with good intentions cannot lie. c.) It is morally right because it can have good effects. d.) It is morally right when it is done to respect another's feelings. what is the impression of the war does Casablanca portray why can't sodium salt be prepared by double decomposition You observe that you see more mockingbirds in small trees and more hawks in large trees. Which of the following is an appropriate scientific question based on this observation?How does the size of a tree affect the bird species that prefer to live in it?How do birds fly?What type of food do birds eat?What time of year to birds mate? explain any 3 factor that affect the mortality rate of nepal Simplify the complex number -16+8. 15.27 The following equilibria were attained at 823 K:COO(s) + H2() Co(s) + H2O(g) K = 67COO(s) + CO(8) = Co(s) + CO2(8) K = 490Based on these equilibria, calculate the equilibrium con-stant for H2(g) + CO2(g) = CO(g) + H2O(g) at 823 K. After the end of the Seven Years' War, the colonists grew much closer to the British, in appreciation for British sacrifices in the war. the British lost complete control of their North American colonies. the British were forced to hand all of North America over to the French. the colonists grew increasingly frustrated with British control and taxes. the French proved to be much better for the colonies after the British left. Which of the following is the best definition of a physical property?A. Tearing paper in half.B. Something that can be observed or measured while changing the identity of the substance.C. The ability of something to undergo a change or reaction.D. Something that can be observed or measured without changing the identity of the substance. Scientists have steadily increased the amount of grain that farms can produce each year. The yield for farms in France is given by y=2.73x2+11000x11000000 where x is the year and y is the grain yield in kilograms per hectare (kg/ha).What does the y-intercept of this function represent? Let f(x)=1/2|x2|. Write a function g whose graph is a translation 7 units to the left of the graph of f.